Finding approximation to stable manifold of saddle point












2












$begingroup$


I am stuck on the following exercise from Strogatz' book on dynamical systems (exercise 6.1.14).




Consider the system $dot{x} = x+e^{-y}, dot{y} = -y$. This system
has a single fixed point, $(-1, 0)$. This is a saddle point. The
unstable manifold is $y=0$, but the stable manifold is some non-linear
curve. Let $(x, y)$ be a point on the stable manifold close to $(-1,
> 0)$
and define $u = x + 1$. Write the stable manifold as $y=a_1u +a_2u^2 + O(u^3)$. To determine the coefficients, derive two
expressions for $dy/du$ and equate them.




I have, as a first try for an equation, simply differentiated $y$ wrt $u$: $frac{dy}{du} = a_1 + 2a_2u + O(u^2)$, where I'm a bit uncertain about the $O(u^2)$, but I suspect we can leave that out anyway, since we're approximating. I'm not sure how we'd find a second equation.



I've found a post that answers the same question, but it ends up with a line, rather than the non-linear curve that is pictured in the book:
enter image description here



Furthermore, I don't see why they have $frac{dy}{du} = frac{dot{y}}{dot{u}}$ or how they calculated the Taylor approximation for $dot{u}$.



Basically, I'm a bit lost, could someone perhaps give me some hints?










share|cite|improve this question









$endgroup$

















    2












    $begingroup$


    I am stuck on the following exercise from Strogatz' book on dynamical systems (exercise 6.1.14).




    Consider the system $dot{x} = x+e^{-y}, dot{y} = -y$. This system
    has a single fixed point, $(-1, 0)$. This is a saddle point. The
    unstable manifold is $y=0$, but the stable manifold is some non-linear
    curve. Let $(x, y)$ be a point on the stable manifold close to $(-1,
    > 0)$
    and define $u = x + 1$. Write the stable manifold as $y=a_1u +a_2u^2 + O(u^3)$. To determine the coefficients, derive two
    expressions for $dy/du$ and equate them.




    I have, as a first try for an equation, simply differentiated $y$ wrt $u$: $frac{dy}{du} = a_1 + 2a_2u + O(u^2)$, where I'm a bit uncertain about the $O(u^2)$, but I suspect we can leave that out anyway, since we're approximating. I'm not sure how we'd find a second equation.



    I've found a post that answers the same question, but it ends up with a line, rather than the non-linear curve that is pictured in the book:
    enter image description here



    Furthermore, I don't see why they have $frac{dy}{du} = frac{dot{y}}{dot{u}}$ or how they calculated the Taylor approximation for $dot{u}$.



    Basically, I'm a bit lost, could someone perhaps give me some hints?










    share|cite|improve this question









    $endgroup$















      2












      2








      2


      2



      $begingroup$


      I am stuck on the following exercise from Strogatz' book on dynamical systems (exercise 6.1.14).




      Consider the system $dot{x} = x+e^{-y}, dot{y} = -y$. This system
      has a single fixed point, $(-1, 0)$. This is a saddle point. The
      unstable manifold is $y=0$, but the stable manifold is some non-linear
      curve. Let $(x, y)$ be a point on the stable manifold close to $(-1,
      > 0)$
      and define $u = x + 1$. Write the stable manifold as $y=a_1u +a_2u^2 + O(u^3)$. To determine the coefficients, derive two
      expressions for $dy/du$ and equate them.




      I have, as a first try for an equation, simply differentiated $y$ wrt $u$: $frac{dy}{du} = a_1 + 2a_2u + O(u^2)$, where I'm a bit uncertain about the $O(u^2)$, but I suspect we can leave that out anyway, since we're approximating. I'm not sure how we'd find a second equation.



      I've found a post that answers the same question, but it ends up with a line, rather than the non-linear curve that is pictured in the book:
      enter image description here



      Furthermore, I don't see why they have $frac{dy}{du} = frac{dot{y}}{dot{u}}$ or how they calculated the Taylor approximation for $dot{u}$.



      Basically, I'm a bit lost, could someone perhaps give me some hints?










      share|cite|improve this question









      $endgroup$




      I am stuck on the following exercise from Strogatz' book on dynamical systems (exercise 6.1.14).




      Consider the system $dot{x} = x+e^{-y}, dot{y} = -y$. This system
      has a single fixed point, $(-1, 0)$. This is a saddle point. The
      unstable manifold is $y=0$, but the stable manifold is some non-linear
      curve. Let $(x, y)$ be a point on the stable manifold close to $(-1,
      > 0)$
      and define $u = x + 1$. Write the stable manifold as $y=a_1u +a_2u^2 + O(u^3)$. To determine the coefficients, derive two
      expressions for $dy/du$ and equate them.




      I have, as a first try for an equation, simply differentiated $y$ wrt $u$: $frac{dy}{du} = a_1 + 2a_2u + O(u^2)$, where I'm a bit uncertain about the $O(u^2)$, but I suspect we can leave that out anyway, since we're approximating. I'm not sure how we'd find a second equation.



      I've found a post that answers the same question, but it ends up with a line, rather than the non-linear curve that is pictured in the book:
      enter image description here



      Furthermore, I don't see why they have $frac{dy}{du} = frac{dot{y}}{dot{u}}$ or how they calculated the Taylor approximation for $dot{u}$.



      Basically, I'm a bit lost, could someone perhaps give me some hints?







      ordinary-differential-equations taylor-expansion dynamical-systems






      share|cite|improve this question













      share|cite|improve this question











      share|cite|improve this question




      share|cite|improve this question










      asked Dec 14 '18 at 21:44









      DashermanDasherman

      1,152818




      1,152818






















          1 Answer
          1






          active

          oldest

          votes


















          2












          $begingroup$

          Remember that



          $$
          frac{{rm d}y}{{rm d}u} = frac{{rm d}y / {rm d}t}{{rm d}u / {rm d}t} tag{1}
          $$



          If you replace



          $$
          frac{{rm d}y}{{rm d}t} = -y tag{2}
          $$



          and



          $$
          frac{{rm d}u}{{rm d}t} = frac{{rm d}x}{{rm d}t} = u - 1 + left(1 - y + frac{y^2}{2} - cdots right) tag{3}
          $$



          In (1) you will get



          $$
          frac{{rm d}y}{{rm d}u} = -frac{y}{u - y + y^2/2 - y^3/6 + cdots} tag{4}
          $$



          Now you replace your expression for $y$: $y(u) = a_1u + a_2 u^2 + cdots$. I'm going to take another path to the solution you just linked, and expand the result to a higher order, you do this by replacing the expression for $y$ in (4) and Taylor expanding it around $u = 0$, the result is



          begin{eqnarray}
          frac{{rm d}y}{{rm d}u} &=& frac{a_1}{a_1 - 1} + frac{a_1^3 - 2 a_2 }{2(a_1 - 1)^2}u + frac{2a_1^4 + a_1^5 - 18a_1^2 a_2 + 12 a_2^2 + 12 a_3 - 12 a_1 a_3}{12(a_1 - 1)^3}u^3 + cdots \
          &=& a_1 + 2a_2 u + 3a_3 u^3 + cdots
          end{eqnarray}



          You solve for coefficients, and you should get



          $$
          a_1 = 2,~~ a_2 = 4/3, ~~a_3 = 10/9
          $$



          The black line below is the result



          enter image description here






          share|cite|improve this answer









          $endgroup$













          • $begingroup$
            Thank you for your answer! How did you find the Taylor expansion around $u=0$? If I substitute the equation for $y(u)$ into $(4)$, I get a function of $u$ that is $0$ for $u=0$, rather than the constant term $frac{a_1}{a_1-1}$ you find.
            $endgroup$
            – Dasherman
            Dec 16 '18 at 14:10








          • 1




            $begingroup$
            @Dasherman It will be zero in the numerator and in the denominator, take the limit when $uto 0$
            $endgroup$
            – caverac
            Dec 16 '18 at 14:17










          • $begingroup$
            Oh, I see. Thank you! I only checked the numerator. Do you also happen to know what the 'correct' way is to treat the $O(u^3)$ term when we do an approximation like this? Should I treat it as $0$ or should I treat it as a term $a_3u^3$?
            $endgroup$
            – Dasherman
            Dec 16 '18 at 14:27






          • 1




            $begingroup$
            @Dasherman It means just to truncate the series at $u^2$, ignore the higher order terms. In my solution above I actually went up to $u^3$ and ignored $u^4$, $u^5$, $cdots$
            $endgroup$
            – caverac
            Dec 16 '18 at 14:35






          • 1




            $begingroup$
            @Dasherman Because if you select the root $a_1 = 0$ you will see that $a_2 = 0$, $a_3 = 0$
            $endgroup$
            – caverac
            Dec 20 '18 at 13:26











          Your Answer





          StackExchange.ifUsing("editor", function () {
          return StackExchange.using("mathjaxEditing", function () {
          StackExchange.MarkdownEditor.creationCallbacks.add(function (editor, postfix) {
          StackExchange.mathjaxEditing.prepareWmdForMathJax(editor, postfix, [["$", "$"], ["\\(","\\)"]]);
          });
          });
          }, "mathjax-editing");

          StackExchange.ready(function() {
          var channelOptions = {
          tags: "".split(" "),
          id: "69"
          };
          initTagRenderer("".split(" "), "".split(" "), channelOptions);

          StackExchange.using("externalEditor", function() {
          // Have to fire editor after snippets, if snippets enabled
          if (StackExchange.settings.snippets.snippetsEnabled) {
          StackExchange.using("snippets", function() {
          createEditor();
          });
          }
          else {
          createEditor();
          }
          });

          function createEditor() {
          StackExchange.prepareEditor({
          heartbeatType: 'answer',
          autoActivateHeartbeat: false,
          convertImagesToLinks: true,
          noModals: true,
          showLowRepImageUploadWarning: true,
          reputationToPostImages: 10,
          bindNavPrevention: true,
          postfix: "",
          imageUploader: {
          brandingHtml: "Powered by u003ca class="icon-imgur-white" href="https://imgur.com/"u003eu003c/au003e",
          contentPolicyHtml: "User contributions licensed under u003ca href="https://creativecommons.org/licenses/by-sa/3.0/"u003ecc by-sa 3.0 with attribution requiredu003c/au003e u003ca href="https://stackoverflow.com/legal/content-policy"u003e(content policy)u003c/au003e",
          allowUrls: true
          },
          noCode: true, onDemand: true,
          discardSelector: ".discard-answer"
          ,immediatelyShowMarkdownHelp:true
          });


          }
          });














          draft saved

          draft discarded


















          StackExchange.ready(
          function () {
          StackExchange.openid.initPostLogin('.new-post-login', 'https%3a%2f%2fmath.stackexchange.com%2fquestions%2f3039914%2ffinding-approximation-to-stable-manifold-of-saddle-point%23new-answer', 'question_page');
          }
          );

          Post as a guest















          Required, but never shown

























          1 Answer
          1






          active

          oldest

          votes








          1 Answer
          1






          active

          oldest

          votes









          active

          oldest

          votes






          active

          oldest

          votes









          2












          $begingroup$

          Remember that



          $$
          frac{{rm d}y}{{rm d}u} = frac{{rm d}y / {rm d}t}{{rm d}u / {rm d}t} tag{1}
          $$



          If you replace



          $$
          frac{{rm d}y}{{rm d}t} = -y tag{2}
          $$



          and



          $$
          frac{{rm d}u}{{rm d}t} = frac{{rm d}x}{{rm d}t} = u - 1 + left(1 - y + frac{y^2}{2} - cdots right) tag{3}
          $$



          In (1) you will get



          $$
          frac{{rm d}y}{{rm d}u} = -frac{y}{u - y + y^2/2 - y^3/6 + cdots} tag{4}
          $$



          Now you replace your expression for $y$: $y(u) = a_1u + a_2 u^2 + cdots$. I'm going to take another path to the solution you just linked, and expand the result to a higher order, you do this by replacing the expression for $y$ in (4) and Taylor expanding it around $u = 0$, the result is



          begin{eqnarray}
          frac{{rm d}y}{{rm d}u} &=& frac{a_1}{a_1 - 1} + frac{a_1^3 - 2 a_2 }{2(a_1 - 1)^2}u + frac{2a_1^4 + a_1^5 - 18a_1^2 a_2 + 12 a_2^2 + 12 a_3 - 12 a_1 a_3}{12(a_1 - 1)^3}u^3 + cdots \
          &=& a_1 + 2a_2 u + 3a_3 u^3 + cdots
          end{eqnarray}



          You solve for coefficients, and you should get



          $$
          a_1 = 2,~~ a_2 = 4/3, ~~a_3 = 10/9
          $$



          The black line below is the result



          enter image description here






          share|cite|improve this answer









          $endgroup$













          • $begingroup$
            Thank you for your answer! How did you find the Taylor expansion around $u=0$? If I substitute the equation for $y(u)$ into $(4)$, I get a function of $u$ that is $0$ for $u=0$, rather than the constant term $frac{a_1}{a_1-1}$ you find.
            $endgroup$
            – Dasherman
            Dec 16 '18 at 14:10








          • 1




            $begingroup$
            @Dasherman It will be zero in the numerator and in the denominator, take the limit when $uto 0$
            $endgroup$
            – caverac
            Dec 16 '18 at 14:17










          • $begingroup$
            Oh, I see. Thank you! I only checked the numerator. Do you also happen to know what the 'correct' way is to treat the $O(u^3)$ term when we do an approximation like this? Should I treat it as $0$ or should I treat it as a term $a_3u^3$?
            $endgroup$
            – Dasherman
            Dec 16 '18 at 14:27






          • 1




            $begingroup$
            @Dasherman It means just to truncate the series at $u^2$, ignore the higher order terms. In my solution above I actually went up to $u^3$ and ignored $u^4$, $u^5$, $cdots$
            $endgroup$
            – caverac
            Dec 16 '18 at 14:35






          • 1




            $begingroup$
            @Dasherman Because if you select the root $a_1 = 0$ you will see that $a_2 = 0$, $a_3 = 0$
            $endgroup$
            – caverac
            Dec 20 '18 at 13:26
















          2












          $begingroup$

          Remember that



          $$
          frac{{rm d}y}{{rm d}u} = frac{{rm d}y / {rm d}t}{{rm d}u / {rm d}t} tag{1}
          $$



          If you replace



          $$
          frac{{rm d}y}{{rm d}t} = -y tag{2}
          $$



          and



          $$
          frac{{rm d}u}{{rm d}t} = frac{{rm d}x}{{rm d}t} = u - 1 + left(1 - y + frac{y^2}{2} - cdots right) tag{3}
          $$



          In (1) you will get



          $$
          frac{{rm d}y}{{rm d}u} = -frac{y}{u - y + y^2/2 - y^3/6 + cdots} tag{4}
          $$



          Now you replace your expression for $y$: $y(u) = a_1u + a_2 u^2 + cdots$. I'm going to take another path to the solution you just linked, and expand the result to a higher order, you do this by replacing the expression for $y$ in (4) and Taylor expanding it around $u = 0$, the result is



          begin{eqnarray}
          frac{{rm d}y}{{rm d}u} &=& frac{a_1}{a_1 - 1} + frac{a_1^3 - 2 a_2 }{2(a_1 - 1)^2}u + frac{2a_1^4 + a_1^5 - 18a_1^2 a_2 + 12 a_2^2 + 12 a_3 - 12 a_1 a_3}{12(a_1 - 1)^3}u^3 + cdots \
          &=& a_1 + 2a_2 u + 3a_3 u^3 + cdots
          end{eqnarray}



          You solve for coefficients, and you should get



          $$
          a_1 = 2,~~ a_2 = 4/3, ~~a_3 = 10/9
          $$



          The black line below is the result



          enter image description here






          share|cite|improve this answer









          $endgroup$













          • $begingroup$
            Thank you for your answer! How did you find the Taylor expansion around $u=0$? If I substitute the equation for $y(u)$ into $(4)$, I get a function of $u$ that is $0$ for $u=0$, rather than the constant term $frac{a_1}{a_1-1}$ you find.
            $endgroup$
            – Dasherman
            Dec 16 '18 at 14:10








          • 1




            $begingroup$
            @Dasherman It will be zero in the numerator and in the denominator, take the limit when $uto 0$
            $endgroup$
            – caverac
            Dec 16 '18 at 14:17










          • $begingroup$
            Oh, I see. Thank you! I only checked the numerator. Do you also happen to know what the 'correct' way is to treat the $O(u^3)$ term when we do an approximation like this? Should I treat it as $0$ or should I treat it as a term $a_3u^3$?
            $endgroup$
            – Dasherman
            Dec 16 '18 at 14:27






          • 1




            $begingroup$
            @Dasherman It means just to truncate the series at $u^2$, ignore the higher order terms. In my solution above I actually went up to $u^3$ and ignored $u^4$, $u^5$, $cdots$
            $endgroup$
            – caverac
            Dec 16 '18 at 14:35






          • 1




            $begingroup$
            @Dasherman Because if you select the root $a_1 = 0$ you will see that $a_2 = 0$, $a_3 = 0$
            $endgroup$
            – caverac
            Dec 20 '18 at 13:26














          2












          2








          2





          $begingroup$

          Remember that



          $$
          frac{{rm d}y}{{rm d}u} = frac{{rm d}y / {rm d}t}{{rm d}u / {rm d}t} tag{1}
          $$



          If you replace



          $$
          frac{{rm d}y}{{rm d}t} = -y tag{2}
          $$



          and



          $$
          frac{{rm d}u}{{rm d}t} = frac{{rm d}x}{{rm d}t} = u - 1 + left(1 - y + frac{y^2}{2} - cdots right) tag{3}
          $$



          In (1) you will get



          $$
          frac{{rm d}y}{{rm d}u} = -frac{y}{u - y + y^2/2 - y^3/6 + cdots} tag{4}
          $$



          Now you replace your expression for $y$: $y(u) = a_1u + a_2 u^2 + cdots$. I'm going to take another path to the solution you just linked, and expand the result to a higher order, you do this by replacing the expression for $y$ in (4) and Taylor expanding it around $u = 0$, the result is



          begin{eqnarray}
          frac{{rm d}y}{{rm d}u} &=& frac{a_1}{a_1 - 1} + frac{a_1^3 - 2 a_2 }{2(a_1 - 1)^2}u + frac{2a_1^4 + a_1^5 - 18a_1^2 a_2 + 12 a_2^2 + 12 a_3 - 12 a_1 a_3}{12(a_1 - 1)^3}u^3 + cdots \
          &=& a_1 + 2a_2 u + 3a_3 u^3 + cdots
          end{eqnarray}



          You solve for coefficients, and you should get



          $$
          a_1 = 2,~~ a_2 = 4/3, ~~a_3 = 10/9
          $$



          The black line below is the result



          enter image description here






          share|cite|improve this answer









          $endgroup$



          Remember that



          $$
          frac{{rm d}y}{{rm d}u} = frac{{rm d}y / {rm d}t}{{rm d}u / {rm d}t} tag{1}
          $$



          If you replace



          $$
          frac{{rm d}y}{{rm d}t} = -y tag{2}
          $$



          and



          $$
          frac{{rm d}u}{{rm d}t} = frac{{rm d}x}{{rm d}t} = u - 1 + left(1 - y + frac{y^2}{2} - cdots right) tag{3}
          $$



          In (1) you will get



          $$
          frac{{rm d}y}{{rm d}u} = -frac{y}{u - y + y^2/2 - y^3/6 + cdots} tag{4}
          $$



          Now you replace your expression for $y$: $y(u) = a_1u + a_2 u^2 + cdots$. I'm going to take another path to the solution you just linked, and expand the result to a higher order, you do this by replacing the expression for $y$ in (4) and Taylor expanding it around $u = 0$, the result is



          begin{eqnarray}
          frac{{rm d}y}{{rm d}u} &=& frac{a_1}{a_1 - 1} + frac{a_1^3 - 2 a_2 }{2(a_1 - 1)^2}u + frac{2a_1^4 + a_1^5 - 18a_1^2 a_2 + 12 a_2^2 + 12 a_3 - 12 a_1 a_3}{12(a_1 - 1)^3}u^3 + cdots \
          &=& a_1 + 2a_2 u + 3a_3 u^3 + cdots
          end{eqnarray}



          You solve for coefficients, and you should get



          $$
          a_1 = 2,~~ a_2 = 4/3, ~~a_3 = 10/9
          $$



          The black line below is the result



          enter image description here







          share|cite|improve this answer












          share|cite|improve this answer



          share|cite|improve this answer










          answered Dec 14 '18 at 22:29









          caveraccaverac

          14.8k31130




          14.8k31130












          • $begingroup$
            Thank you for your answer! How did you find the Taylor expansion around $u=0$? If I substitute the equation for $y(u)$ into $(4)$, I get a function of $u$ that is $0$ for $u=0$, rather than the constant term $frac{a_1}{a_1-1}$ you find.
            $endgroup$
            – Dasherman
            Dec 16 '18 at 14:10








          • 1




            $begingroup$
            @Dasherman It will be zero in the numerator and in the denominator, take the limit when $uto 0$
            $endgroup$
            – caverac
            Dec 16 '18 at 14:17










          • $begingroup$
            Oh, I see. Thank you! I only checked the numerator. Do you also happen to know what the 'correct' way is to treat the $O(u^3)$ term when we do an approximation like this? Should I treat it as $0$ or should I treat it as a term $a_3u^3$?
            $endgroup$
            – Dasherman
            Dec 16 '18 at 14:27






          • 1




            $begingroup$
            @Dasherman It means just to truncate the series at $u^2$, ignore the higher order terms. In my solution above I actually went up to $u^3$ and ignored $u^4$, $u^5$, $cdots$
            $endgroup$
            – caverac
            Dec 16 '18 at 14:35






          • 1




            $begingroup$
            @Dasherman Because if you select the root $a_1 = 0$ you will see that $a_2 = 0$, $a_3 = 0$
            $endgroup$
            – caverac
            Dec 20 '18 at 13:26


















          • $begingroup$
            Thank you for your answer! How did you find the Taylor expansion around $u=0$? If I substitute the equation for $y(u)$ into $(4)$, I get a function of $u$ that is $0$ for $u=0$, rather than the constant term $frac{a_1}{a_1-1}$ you find.
            $endgroup$
            – Dasherman
            Dec 16 '18 at 14:10








          • 1




            $begingroup$
            @Dasherman It will be zero in the numerator and in the denominator, take the limit when $uto 0$
            $endgroup$
            – caverac
            Dec 16 '18 at 14:17










          • $begingroup$
            Oh, I see. Thank you! I only checked the numerator. Do you also happen to know what the 'correct' way is to treat the $O(u^3)$ term when we do an approximation like this? Should I treat it as $0$ or should I treat it as a term $a_3u^3$?
            $endgroup$
            – Dasherman
            Dec 16 '18 at 14:27






          • 1




            $begingroup$
            @Dasherman It means just to truncate the series at $u^2$, ignore the higher order terms. In my solution above I actually went up to $u^3$ and ignored $u^4$, $u^5$, $cdots$
            $endgroup$
            – caverac
            Dec 16 '18 at 14:35






          • 1




            $begingroup$
            @Dasherman Because if you select the root $a_1 = 0$ you will see that $a_2 = 0$, $a_3 = 0$
            $endgroup$
            – caverac
            Dec 20 '18 at 13:26
















          $begingroup$
          Thank you for your answer! How did you find the Taylor expansion around $u=0$? If I substitute the equation for $y(u)$ into $(4)$, I get a function of $u$ that is $0$ for $u=0$, rather than the constant term $frac{a_1}{a_1-1}$ you find.
          $endgroup$
          – Dasherman
          Dec 16 '18 at 14:10






          $begingroup$
          Thank you for your answer! How did you find the Taylor expansion around $u=0$? If I substitute the equation for $y(u)$ into $(4)$, I get a function of $u$ that is $0$ for $u=0$, rather than the constant term $frac{a_1}{a_1-1}$ you find.
          $endgroup$
          – Dasherman
          Dec 16 '18 at 14:10






          1




          1




          $begingroup$
          @Dasherman It will be zero in the numerator and in the denominator, take the limit when $uto 0$
          $endgroup$
          – caverac
          Dec 16 '18 at 14:17




          $begingroup$
          @Dasherman It will be zero in the numerator and in the denominator, take the limit when $uto 0$
          $endgroup$
          – caverac
          Dec 16 '18 at 14:17












          $begingroup$
          Oh, I see. Thank you! I only checked the numerator. Do you also happen to know what the 'correct' way is to treat the $O(u^3)$ term when we do an approximation like this? Should I treat it as $0$ or should I treat it as a term $a_3u^3$?
          $endgroup$
          – Dasherman
          Dec 16 '18 at 14:27




          $begingroup$
          Oh, I see. Thank you! I only checked the numerator. Do you also happen to know what the 'correct' way is to treat the $O(u^3)$ term when we do an approximation like this? Should I treat it as $0$ or should I treat it as a term $a_3u^3$?
          $endgroup$
          – Dasherman
          Dec 16 '18 at 14:27




          1




          1




          $begingroup$
          @Dasherman It means just to truncate the series at $u^2$, ignore the higher order terms. In my solution above I actually went up to $u^3$ and ignored $u^4$, $u^5$, $cdots$
          $endgroup$
          – caverac
          Dec 16 '18 at 14:35




          $begingroup$
          @Dasherman It means just to truncate the series at $u^2$, ignore the higher order terms. In my solution above I actually went up to $u^3$ and ignored $u^4$, $u^5$, $cdots$
          $endgroup$
          – caverac
          Dec 16 '18 at 14:35




          1




          1




          $begingroup$
          @Dasherman Because if you select the root $a_1 = 0$ you will see that $a_2 = 0$, $a_3 = 0$
          $endgroup$
          – caverac
          Dec 20 '18 at 13:26




          $begingroup$
          @Dasherman Because if you select the root $a_1 = 0$ you will see that $a_2 = 0$, $a_3 = 0$
          $endgroup$
          – caverac
          Dec 20 '18 at 13:26


















          draft saved

          draft discarded




















































          Thanks for contributing an answer to Mathematics Stack Exchange!


          • Please be sure to answer the question. Provide details and share your research!

          But avoid



          • Asking for help, clarification, or responding to other answers.

          • Making statements based on opinion; back them up with references or personal experience.


          Use MathJax to format equations. MathJax reference.


          To learn more, see our tips on writing great answers.




          draft saved


          draft discarded














          StackExchange.ready(
          function () {
          StackExchange.openid.initPostLogin('.new-post-login', 'https%3a%2f%2fmath.stackexchange.com%2fquestions%2f3039914%2ffinding-approximation-to-stable-manifold-of-saddle-point%23new-answer', 'question_page');
          }
          );

          Post as a guest















          Required, but never shown





















































          Required, but never shown














          Required, but never shown












          Required, but never shown







          Required, but never shown

































          Required, but never shown














          Required, but never shown












          Required, but never shown







          Required, but never shown







          Popular posts from this blog

          Plaza Victoria

          In PowerPoint, is there a keyboard shortcut for bulleted / numbered list?

          How to put 3 figures in Latex with 2 figures side by side and 1 below these side by side images but in...